Last visit was: 23 Apr 2024, 16:11 It is currently 23 Apr 2024, 16:11

Close
GMAT Club Daily Prep
Thank you for using the timer - this advanced tool can estimate your performance and suggest more practice questions. We have subscribed you to Daily Prep Questions via email.

Customized
for You

we will pick new questions that match your level based on your Timer History

Track
Your Progress

every week, we’ll send you an estimated GMAT score based on your performance

Practice
Pays

we will pick new questions that match your level based on your Timer History
Not interested in getting valuable practice questions and articles delivered to your email? No problem, unsubscribe here.
Close
Request Expert Reply
Confirm Cancel
SORT BY:
Date
Tags:
Difficulty: 555-605 Levelx   Assumptionx                     
Show Tags
Hide Tags
User avatar
Director
Director
Joined: 18 May 2008
Posts: 696
Own Kudos [?]: 2798 [441]
Given Kudos: 0
Send PM
Most Helpful Reply
e-GMAT Representative
Joined: 02 Nov 2011
Posts: 4341
Own Kudos [?]: 30776 [87]
Given Kudos: 632
GMAT Date: 08-19-2020
Send PM
GMAT Club Verbal Expert
Joined: 13 Aug 2009
Status: GMAT/GRE/LSAT tutors
Posts: 6917
Own Kudos [?]: 63649 [84]
Given Kudos: 1773
Location: United States (CO)
GMAT 1: 780 Q51 V46
GMAT 2: 800 Q51 V51
GRE 1: Q170 V170

GRE 2: Q170 V170
Send PM
Tutor
Joined: 16 Oct 2010
Posts: 14816
Own Kudos [?]: 64882 [2]
Given Kudos: 426
Location: Pune, India
Send PM
A researcher discovered that people who have low levels of immune-syst [#permalink]
1
Kudos
1
Bookmarks
Expert Reply
ritula wrote:
A researcher discovered that people who have low levels of immune-system activity tend to score much lower on tests of mental health than do people with normal or high immune-system activity. The researcher concluded from this experiment that the immune system protects against mental illness as well as against physical disease.

The researcher’s conclusion depends on which of the following assumptions?


(A) High immune-system activity protects against mental illness better than normal immune-system activity does.

(B) Mental illness is similar to physical disease in its effects on body systems.

(C) People with high immune-system activity cannot develop mental illness.

(D) Mental illness does not cause people’s immune-system activity to decrease.

(E) Psychological treatment of mental illness is not as effective as is medical treatment.



Research: People with lower level of immune systems score lower on mental tests.

Conclusion: Immune system protects against mental illness too.

The conclusion is derived because a link was found between immune system and mental health. Lower the immune system, lower mental health. Better the immune system, better mental health.
So it was concluded that immune system protects mental health too (along with physical health which we anyway know it protects). Because immune system is low, it causes mental illness. If immune system were better, mental illness would not have been there.

What is the assumption here? That the causation is not the other way around - that it is not mental disturbance that is causing lower immune system. What if mental illness were causing immune system to be low? Then our conclusion does not hold.

(A) High immune-system activity protects against mental illness better than normal immune-system activity does.

High vs normal comparison is irrelevant.

(B) Mental illness is similar to physical disease in its effects on body systems.

Not an assumption. We don't need mental illness to be similar to physical disease. They could be very different from each other (and have very different effects on body's systems) and still immune system could be responsible for the health of both.

(C) People with high immune-system activity cannot develop mental illness.

Not an assumption.

(D) Mental illness does not cause people’s immune-system activity to decrease.

Correct. As per our discussion, we said that to say that lower immune system causes mental illness, we are assuming that mental illness does not cause lower immune system.
A and B co-exist.
We are concluding that A is causing B.
Then we are assuming that it is not B that is causing A.

(E) Psychological treatment of mental illness is not as effective as is medical treatment.

Irrelevant
General Discussion
User avatar
Manager
Manager
Joined: 21 Mar 2008
Posts: 121
Own Kudos [?]: 68 [14]
Given Kudos: 0
Send PM
Re: A researcher discovered that people who have low levels of immune-syst [#permalink]
8
Kudos
6
Bookmarks
ritula wrote:
A researcher discovered that people who have low levels of immune-system activity tend to score much lower on tests of mental health than do people with normal or high immune-system activity. The researcher concluded from this experiment that the immune system protects against mental illness as well as against physical disease.
The researcher’s conclusion depends on which of the following assumptions?
A. High immune-system activity protects against mental illness better than normal immune-system activity
does.
B. Mental illness is similar to physical disease in its effects on body systems.
C. People with high immune-system activity cannot develop mental illness.
D. Mental illness does not cause people’s immune-system activity to decrease.
E. Psychological treatment of mental illness is not as effective as is medical treatment.


D
The author concludes that "the immune system protects against mental illness as well as against physical disease", So the implication is that better immune system protects people from mental/physical illnesses. The author must assume that the reversal is not true that mental illness is not the cause of decreased immune system activity in the first place.
Negate this assumption and the argument falls apart.

Hope this helps.
Senior Manager
Senior Manager
Joined: 26 Nov 2009
Status:Impossible is not a fact. It's an opinion. It's a dare. Impossible is nothing.
Affiliations: University of Chicago Booth School of Business
Posts: 470
Own Kudos [?]: 2377 [6]
Given Kudos: 36
Location: Singapore
Concentration: General Management, Finance
Schools: Chicago Booth - Class of 2015
Send PM
Re: A researcher discovered that people who have low levels of immune-syst [#permalink]
4
Kudos
2
Bookmarks
Conclusion : Immune system protects against mental illness. X = immune system Y = protection against mental illness
The most important assumption in the causal argument is the preclusion from reversal of causality. i.e. Y does not leads to X. D does that.

D. Mental illness does not cause people’s immune-system activity to decrease. ---> Y does NOT leads to X.

Hope this is clear.
User avatar
Manager
Manager
Joined: 11 May 2010
Posts: 109
Own Kudos [?]: 548 [18]
Given Kudos: 11
Send PM
Re: A researcher discovered that people who have low levels of immune-syst [#permalink]
15
Kudos
3
Bookmarks
D it is.

This is the classic problem of Defender Assumption.

It usually happens when two events A and B happen at the same time and Researcher then conclude that one event lead to the event say A leads to B. Of course then the assumption has to be that B doesn't lead to A. If B leads to A, how can you say A leads to B.
avatar
Intern
Intern
Joined: 17 Jun 2012
Posts: 2
Own Kudos [?]: 2 [1]
Given Kudos: 0
Send PM
Re: A researcher discovered that people who have low levels of immune-syst [#permalink]
1
Kudos
Why not B? If B is negated then also the argument falls flat..also if we dont use negation technique, option B is more apt than option D (IMO)

Posted from my mobile device
User avatar
Manhattan Prep Instructor
Joined: 30 Apr 2012
Posts: 782
Own Kudos [?]: 2583 [7]
Given Kudos: 5
Send PM
Re: A researcher discovered that people who have low levels of immune-syst [#permalink]
7
Kudos
Expert Reply
B is a very tempting answer here because the conclusion bundles together mental health and physical disease. This problem type, however, is a very specific CR assumption subtype. In this subtype, the argument will present two factors that demonstrate correlation - in this case Immune System and Mental Illness. The conclusion asserts a causation (without any rationale) from one factor to the next - here the argument concludes that the Immune System protects against Mental Illness. The implicit assumption in that line of reasoning is that the causation does NOT go in the other direction or in other words you have to assume that Mental Illness does not impact the immune system. Answer choice D states that assumption correctly.

KW
avatar
Intern
Intern
Joined: 28 Apr 2013
Posts: 1
Own Kudos [?]: [3]
Given Kudos: 0
Send PM
A researcher discovered that people who have low levels of immune-syst [#permalink]
3
Kudos
ritula wrote:
A researcher discovered that people who have low levels of immune-system activity tend to score much
lower on tests of mental health than do people with normal or high immune-system activity. The researcher
concluded from this experiment that the immune system protects against mental illness as well as against
physical disease.
The researcher’s conclusion depends on which of the following assumptions?
A. High immune-system activity protects against mental illness better than normal immune-system activity
does.
B. Mental illness is similar to physical disease in its effects on body systems.
C. People with high immune-system activity cannot develop mental illness.
D. Mental illness does not cause people’s immune-system activity to decrease.
E. Psychological treatment of mental illness is not as effective as is medical treatment.


I picked C.I am not finding why c is wrong.Can u explain why c is wrong?
e-GMAT Representative
Joined: 02 Nov 2011
Posts: 4341
Own Kudos [?]: 30776 [16]
Given Kudos: 632
GMAT Date: 08-19-2020
Send PM
Re: A researcher discovered that people who have low levels of immune-syst [#permalink]
12
Kudos
4
Bookmarks
Expert Reply
supri23 wrote:
ritula wrote:
A researcher discovered that people who have low levels of immune-system activity tend to score much
lower on tests of mental health than do people with normal or high immune-system activity. The researcher
concluded from this experiment that the immune system protects against mental illness as well as against
physical disease.
The researcher’s conclusion depends on which of the following assumptions?
A. High immune-system activity protects against mental illness better than normal immune-system activity
does.
B. Mental illness is similar to physical disease in its effects on body systems.
C. People with high immune-system activity cannot develop mental illness.
D. Mental illness does not cause people’s immune-system activity to decrease.
E. Psychological treatment of mental illness is not as effective as is medical treatment.


I picked C.I am not finding why c is wrong.Can u explain why c is wrong?


Hi,

To tackle your doubt, let me begin from the passage itself.

Understanding the Passage

A researcher discovered that people who have low levels of immune-system activity tend to score much lower on tests of mental health than do people with normal or high immune-system activity.

This statement talks about a discovery by a researcher. The researcher discovered that people who have

low immune system activity have lower mental health
normal or high immune system activity have better mental health.

So, what he saw was that mental health increased with increase in immune system activity.

The researcher concluded from this experiment that the immune system protects against mental illness as well as against physical disease.

The researcher thought about the reason as to why this pattern exists. He thought an explanation for this pattern is that immune system protects against mental illness or poor mental health. If his explanation is true, then people with lower immune system should have poorer mental health than people with normal or higher immune system activity. This is what he has observed. So, he made the conclusion as stated in the above statement of the passage.

Pre-thinking Assumption

Now, what is the assumption built in the conclusion drawn by the researcher?

The assumption is that there is no other explanation which could explain the given observed pattern. If there are other explanations for the observed pattern, then it would cast a serious doubt on the conclusion drawn by the author.

Now, what could be an alternate explanation?

This could be a hard question for people new to causal arguments but for people who have done some practice of causal arguments, they can figure out the answer to this quite easily.

Remember, the researchers concluded that immune system protects against mental illness because this can explain the observed pattern. Now, if we say that mental illness causes decline in immune system activity, then even this statement could explain the given observed pattern (where both immune system activity and mental health increase or decrease simultaneously). Now, if this could also be an explanation, then the author must have considered this explanation and 'assumed' that this cannot be be true.

So, an assumption in the above argument is that mental illness does not cause people’s immune-system activity to decrease. This is what option D is.

Now, coming to option C:

People with high immune-system activity cannot develop mental illness.

First of all, we need to think why does the author need to assume this? Remember a golden rule:

The author assumes only those things without which his argument will not hold true.

For this reason, the 'assumptions' are called must be true statements, which means that the assumptions must be true for the conclusion to hold true.

Now, is Option C a must be true statement?

What if option C is false i.e. People with high immune-system activity can develop mental illness. Will it falsify the conclusion?

The answer is No. The conclusion can still hold even when this statement is false. Therefore, it cannot be an assumption.

Hope this helps :)

Thanks,
Chiranjeev
User avatar
Intern
Intern
Joined: 14 Apr 2013
Posts: 36
Own Kudos [?]: 199 [3]
Given Kudos: 3
GMAT 1: 740 Q50 V40
Send PM
Re: A researcher discovered that people who have low levels of immune-syst [#permalink]
2
Kudos
1
Bookmarks
Argument:
Researcher discovery from experiment: people with low levels of immune-system activity tend to score much lower on tests of mental health than do people with normal or high immune-system activity.
Researcher conclusion: the immune system protects against mental illness as well as against physical disease.

Pre thinking:
Researcher observation is that low levels of immune-system activity tend to have lower score on mental health.
He made the conclusion that immune system protect against mental illness.
We can make another conclusion that lower score on mental health (i.e. mental) illness causes lower immune system activity. So an underlying assumption here is that mental illness does not cause low immune system activity.

Analysis of answer Choices:
(A) High immune-system activity protects against mental illness better than normal immune-system activity does.
INCORRECT: the argument is not about high and normal immune system. Nothing about normal immune system is provided.

(B) Mental illness is similar to physical disease in its effects on body systems.
INCORRECT: The argument is focussed on mental illness and immune system and conclusion has nothing to do with body systems in general.

(C) People with high immune-system activity cannot develop mental illness.
INCORRECT: Conclusion says that immune system protects against mental illness like it does again physical illness. Doesn’t mean that people with high immune system can’t develop mental or physical illness.

(D) Mental illness does not cause people's immune-system activity to decrease.
CORRECT: Look at the pre thinking. If this is false then we will get the conclusion that lower score on mental health (i.e. mental illness) causes lower immune system activity. So for researchers conclusion to hold this must be true and be the assumption.

(E)Psychological treatment of mental illness is not as effective as is medical treatment.
INCORRECT: This doesn’t tell if immune system is effective for preventing mental illness or not.

The key here was that researcher could have made multiple conclusion from his experiment.Why he reached at one conclusion helped me in determining the underlying assumption.
Hope this helps.
User avatar
Manhattan Prep Instructor
Joined: 30 Apr 2012
Posts: 782
Own Kudos [?]: 2583 [6]
Given Kudos: 5
Send PM
Re: A researcher discovered that people who have low levels of immune-syst [#permalink]
6
Kudos
Expert Reply
TGC-

In your previous post you mentioned the key without realizing it. The premise discusses the connection between the immune system and mental illness. You said "Nowhere the argument mentioned PD (Physical Disease) in the premises". Physical Disease is only in the conclusion. Assumptions are the logical gaps between Premises and the Conclusion. Since Physical Disease isn't mentioned in the premises, there isn't a logical gap between premise and conclusion related to Physical Disease. Really, Physical Disease in the conclusion is acting as a distractor in the question. You could read the conclusion like this: "..the immune system protects against mental illness (just like it protects against physical disease)". The core of the conclusion is immune system PROTECTING against mental illness.

Now, let's explore option B just a bit to get fully comfortable that it's not the assumption. B states "Mental illness is similar to physical disease in its effects on body systems." Remember that assumptions are the logical gaps between premises and the conclusion.

Premise: People with low immune system score lower on mental health tests (low immune system and low mental health show up together)
Assumption - GAP
Conclusion: Therefore the immune system PROTECTS against mental health (like it PROTECTS against physical disease)

Is there a logical gap involving the effects? Do we have to assume that the EFFECTS of mental illness have to be the same as the EFFECTS of physical disease to assume that the immune system PROTECTS against mental health? No - the effects are out of the scope of the argument. We are concerned about immune system protecting against mental health problems. The immune system can PROTECT against two things that have very different effects (for example - my front door protects my home & family from the weather and from burglars, which have very different effects on my home & family).

Remember to look at the GAP between premise and conclusion. Premise says that IS & MH happen together & the conclusion states that the immune system PROTECTS against mental health. To believe that conclusion we have to assume that bad mental health doesn't impact the immune system (choice D)

KW
User avatar
Manhattan Prep Instructor
Joined: 30 Apr 2012
Posts: 782
Own Kudos [?]: 2583 [4]
Given Kudos: 5
Send PM
Re: A researcher discovered that people who have low levels of immune-syst [#permalink]
3
Kudos
1
Bookmarks
Expert Reply
I'm responding to a PM on this and thought the response would be valuable for all...

"Hi,
I agree that B may not be the right choice, But can you explain why [C] cannot be right? Here is my line of thought:
-C.People with High Immune system cannot develop mental illness.
As taught, let us assume this is true. If yes, it establishes a direct relation between Mental Illness and Immune system. Incase we negate it [i.e.-People w/ high immunity CAN be mentally ill], the Authors conclusion breaks apart. However, people say that this answer choice is "really strong and unfounded".
However, the same can be said for answer D too. Also, I think answer D weakens the authors argument as if we are to assume mental illness does not cause low immunity, it does not support the conclusion that 'Immune sytem protects agains both physical and mental illness'...
Kindly explain."

People are right when they say that C is "really strong". C goes beyond what was concluded by the argument. The conclusion says that the immune system PROTECTS against mental illness as well as against physical disease. We know that the protection provided by the immune system against physical disease is not without some faults (we all get sick, right?). Therefore, the conclusion is stating that the immune system is REDUCING the incidence or severity of mental illness, not that mental illness cannot develop. Answer choice C goes beyond the conclusion and therefore cannot be the GAP between the premise and the conclusion.

Now, I'll talk about answer choice D. It's a recognizable pattern on GMAT CR problems that goes back to correlation vs. causation. The argument's premise is that there is a correlation between the immune system and mental illness. Then in the conclusion the author is moving from a "correlation" to an actual causation and that it is the immune system that is "causing" the difference in mental illness. Nothing in the argument is stating that it's the immune system that is impacting mental health, so we have to assume that the causation is not in the other direction - Mental Illness does not cause changes in the immune system.

KW
avatar
Intern
Intern
Joined: 26 Feb 2014
Posts: 1
Own Kudos [?]: 1 [1]
Given Kudos: 0
Send PM
Re: A researcher discovered that people who have low levels of immune-syst [#permalink]
1
Kudos
KyleWiddison wrote:
B is a very tempting answer here because the conclusion bundles together mental health and physical disease. This problem type, however, is a very specific CR assumption subtype. In this subtype, the argument will present two factors that demonstrate correlation - in this case Immune System and Mental Illness. The conclusion asserts a causation (without any rationale) from one factor to the next - here the argument concludes that the Immune System protects against Mental Illness. The implicit assumption in that line of reasoning is that the causation does NOT go in the other direction or in other words you have to assume that Mental Illness does not impact the immune system. Answer choice D states that assumption correctly.

KW

But why the author mentioned about "physical disease"? It's really confusing
Thx.
User avatar
Manhattan Prep Instructor
Joined: 30 Apr 2012
Posts: 782
Own Kudos [?]: 2583 [1]
Given Kudos: 5
Send PM
Re: A researcher discovered that people who have low levels of immune-syst [#permalink]
1
Kudos
Expert Reply
boris54 wrote:
But why the author mentioned about "physical disease"? It's really confusing
Thx.


That is precisely why physical disease is added - to confuse you. It is a distractor from the core issue of the argument...the direction of the causation.

KW
avatar
Manager
Manager
Joined: 02 Mar 2012
Posts: 201
Own Kudos [?]: 292 [4]
Given Kudos: 4
Schools: Schulich '16
Send PM
Re: A researcher discovered that people who have low levels of immune-syst [#permalink]
2
Kudos
2
Bookmarks
D is the answer.

if you guys have read POWERSCORE CR bible then you are going to understand what i am writing below.

this type of assumption questions are DEFENDER type. in these type of questions the answer choice will try to defend the argument conclusion.

the conclusion here in the argument is:

The researcher concluded from this experiment that the immune system protects against mental illness as well as against
physical disease.

or in short

immune system -->> protects against mental illness and physical too.


option D read as:

D. Mental illness does not cause people’s immune-system activity to decrease.

this statement defends the argument that the reverse is not true( its defending the argument)

hope it helps
Tutor
Joined: 15 Nov 2013
Posts: 97
Own Kudos [?]: 925 [4]
Given Kudos: 3
GMAT 1: 800 Q51 V51
GRE 1: Q170 V170

GRE 2: Q170 V170
Send PM
Re: A researcher discovered that people who have low levels of immune-syst [#permalink]
4
Kudos
Expert Reply
I received a private communication about this problem.

There's an unusual twist in the stated conclusion here that's definitely worth noticing.
This is the stated conclusion:
The researcher concluded from this experiment that the immune system protects against mental illness as well as against physical disease.

AT FIRST this conclusion may seem to have two parts:
• The immune system protects against mental illness.
• The immune system protects against physical disease.

A look over the argument, though, reveals that there are NO premises related to physical disease. Therefore, the ACTUAL conclusion SUPPORTED by this argument is
"the immune system protects against mental illness".


The second part — about physical disease — is just a piece of universal everyday knowledge. It is NOT supported by the argument, and so, therefore, it is NOT part of the argument's conclusion.

I haven't seen too many concluding statements structured like this — but it's worth noting that they exist, both on GMAT CR and in real life.
Tutor
Joined: 15 Nov 2013
Posts: 97
Own Kudos [?]: 925 [2]
Given Kudos: 3
GMAT 1: 800 Q51 V51
GRE 1: Q170 V170

GRE 2: Q170 V170
Send PM
Re: A researcher discovered that people who have low levels of immune-syst [#permalink]
2
Kudos
Expert Reply
RonPurewal wrote:
I haven't seen too many concluding statements structured like this — but it's worth noting that they exist, both on GMAT CR and in real life.


As an example of the latter ("in real life")—

Imagine that a website or magazine carries a recipe for a vegetable dish, describing the dish as "delicious as well as nutritious!"

This statement won't be the conclusion of an argument — i.e., it's VERY unlikely that any premises would be given to support these statements; like other statements of opinion, they would just be declared — but its two parts work like those of the sentence discussed here:
• "Delicious" is the ACTUAL POINT of the sentence;
• "Nutritious" is just something most people already accept and believe about vegetable dishes in general (and is mentioned, presumably, because readers might find it surprising that a "nutritious" dish could also be "delicious!").
VP
VP
Joined: 14 Aug 2019
Posts: 1378
Own Kudos [?]: 846 [0]
Given Kudos: 381
Location: Hong Kong
Concentration: Strategy, Marketing
GMAT 1: 650 Q49 V29
GPA: 3.81
Send PM
Re: A researcher discovered that people who have low levels of immune-syst [#permalink]
Quote:
A researcher discovered that people who have low levels of immune-system activity tend to score much lower on tests of mental health than do people with normal or high immune-system activity. The researcher concluded from this experiment that the immune system protects against mental illness as well as against physical disease.

The researcher’s conclusion depends on which of the following assumptions?


Hi AndrewN sir VeritasKarishma

Could you give comment on my approach of handling assumption question. It will help me to build skills on this base. Your comments would be appreciable. Please add suggestions.

Approach:
Step 1: Understand the meaning of option
Step2: Check impact on Conclusion
Step3: If does not WEAKEN, try negate.
Step 4: If breaks down the reasoning of conclusion then it is an assumption


Quote:
(A) High immune-system activity protects against mental illness better than normal immune-system activity does.

1. It means immune system activities protects against mental illness.
2. It means immune system plays an important role in protection. It doesn’t weaken our conclusion.
3. Negate: high Immune system activity doesn’t protect better than normal immunity system activities against mental illness.
4. Our conclusion can still hold be true that immune system protects against mental illness as well against physical disease.
Hence it is not an assumption.

Quote:
(B) Mental illness is similar to physical disease in its effects on body systems.

1. Meaning: Mental Illness is similar to physical disease in ITS EFFECTS ON BODY SYSTEMS.
2. How doesn’t affect matter to conclusion that immune system protects against mental illness as well as against physical disease.
Hence it has no impact on conclusion. So we no need to do any negation.

Quote:
(C) People with high immune-system activity cannot develop mental illness.

1. It means high immune system activity plays an important role against mental illness.
2. It doesn’t weaken the claim that immune system can protect against mental illness and physical disease.
3. Negate: High immune system can develop mental illness.
Impact on conclusion: It doesn’t impact whether immune system can still protect against mental illness and physical disease.
4. Since it doesn’t break down our conclusion. Hence it is not an assumption.

Quote:
(D) Mental illness does not cause people’s immune-system activity to decrease.

Meaning: Mental illness doesn’t cause people immune system activity to decrease.
2. It means immune system may or may not have some role to play . Lets negate to see the impact.
3. negate: mental illness cause people immunce system to decrease
4. break down our conclusion : if mental illness cause people immune system to decrease then it immune system doesn’t protect against mental illness
-> It breaks down our conclusion. Hence it is assumption.


Quote:
(E) Psychological treatment of mental illness is not as effective as is medical treatment.

1. It talks about one treatment vs another treatment.
2. It doesn’t impact our conclusion that immune system protects against mental illness.
-> So it is not a strengthener. So we no need to do any negation. Hence it cannot be an assumption.

Thanks!
GMAT Club Bot
Re: A researcher discovered that people who have low levels of immune-syst [#permalink]
 1   2   
Moderators:
GMAT Club Verbal Expert
6917 posts
GMAT Club Verbal Expert
238 posts
CR Forum Moderator
832 posts

Powered by phpBB © phpBB Group | Emoji artwork provided by EmojiOne